2018 AMC 10A Problems/Problem 7

Revision as of 14:54, 8 February 2018 by Ishankhare (talk | contribs) (Created page with "For how many (not necessarily positive) integer values of <math>n</math> is the value of <math>4000\cdot \left(\tfrac{2}{5}\right)^n</math> an integer? <math> \textbf{(A) }3...")
(diff) ← Older revision | Latest revision (diff) | Newer revision → (diff)

For how many (not necessarily positive) integer values of $n$ is the value of $4000\cdot \left(\tfrac{2}{5}\right)^n$ an integer?

$\textbf{(A) }3 \qquad \textbf{(B) }4 \qquad \textbf{(C) }6 \qquad \textbf{(D) }8 \qquad \textbf{(E) }9 \qquad$